subject
Mathematics, 18.07.2019 13:30 awesomegrill

Let $$f(x) = \frac{x^2}{x^2 - 1}.$$find the largest integer $n$ so that $f(2) \cdot f(3) \cdot f(4) \cdots f(n-1) \cdot f(n) < 1.98.$

ansver
Answers: 1

Another question on Mathematics

question
Mathematics, 21.06.2019 15:00
Write the product as a trinomial. (x + 2)(x + 3) x2 + 5x + 6 x2 + 6x + 6 x2 + 6x + 5 x2 + 5
Answers: 2
question
Mathematics, 21.06.2019 19:30
Write the expression 7x^3-28xy^2 as the product of 3 factors
Answers: 2
question
Mathematics, 21.06.2019 23:10
Pls yall i really need the answer fr fr
Answers: 1
question
Mathematics, 22.06.2019 00:10
Write the slope intercept equation for line ab.
Answers: 2
You know the right answer?
Let $$f(x) = \frac{x^2}{x^2 - 1}.$$find the largest integer $n$ so that $f(2) \cdot f(3) \cdot f(4)...
Questions
question
World Languages, 22.09.2021 14:00
question
Biology, 22.09.2021 14:00
question
Mathematics, 22.09.2021 14:00
question
Mathematics, 22.09.2021 14:00
Questions on the website: 13722367